You are on page 1of 2

ACGA - Ejercicio 1.7.

Javier Linares Torres


Curso 2018-2019

Ejercicio 1.7. Sean I1 , . . . , Ir ideales de un anillo A tales que I1 ∩ · · · ∩ Ir = {0}


y cada cociente A/Ii para i = 1, . . . , r es noetheriano. Probar que A es noetheriano.

Algunas observaciones previas antes de dar la solución al ejercicio:

1. Un anillo A es notheriano si y sólo si A, visto como A-módulo, es un A-módulo


noetheriano, i.e., todo submódulo es finitamente generado, ya que los submódulos de
A son precisamente sus ideales.

2. Todo submódulo N de un A-módulo M noetheriano es noetheriano pues los submó-


dulos de N son también submódulos de M .

3. Podemos ver cada cociente A/Ii para i = 1, . . . r como un A-módulo, definiendo la


acción de A sobre A/Ii como x(a + Ii ) = xa + Ii para todo x ∈ A y a + Ii ∈ A/Ii . De
hecho, A/Ii es un A-módulo noetheriano, pues dado cualquier submódulo N ⊆ A/Ii ,
al estar N finitamente generado como A/Ii -módulo también lo está como A-módulo.

4. Veamos que la suma directa de un número finito de A-módulos noetherianos es noet-


heriano [AM73, Cor 6.4.]. Para ello, en primer lugar hemos de notar que un A-módulo
M es noetheriano si y sólo si M cumple la condición de cadena ascendente. Ahora
necesitamos el siguiente lema

Lema 1. Sea
β
0 → M ′ → M → M ′′ → 0
α

una sucesión exacta de A-módulos. Entonces M es noetheriano si y solo si M ′ y M ′′


son notherianos.

Prueba.

(⇒) Sea (Ni′ )i≥1 una cadena ascendente de submódulos de M ′ . Entonces (α(Ni′ ))i≥1
es una cadena ascendente de submódulos de M que debe estabilizarse. Dado que
α es inyectiva, la cadena (Ni′ )i≥1 también es estacionaria. De manera análoga,
una cadena ascendente (Ni′′ )i≥1 de submódulos de M ′′ da lugar a otra cade-
( )
na ascendente β −1 (Ni′′ ) i≥1 en M que debe ser estacionaria y dado que β es
sobreyectiva, (Ni′′ )i≥1 también es estacionaria.
( )
(⇐) Sea (Ni )i≥1 una cadena ascendente de submódulos de M ; entonces α−1 (Ni ) i≥1
es una cadena en M ′ , y (β(Ni ))i≥1 es una cadena en M ′′ . Para i suficientemente
grande, ambas cadenas son estacionarias. Falta ver que si N1 ⊆ N2 son submó-
dulos de M entonces α−1 (N1 ) = α−1 (N2 ) y β(N1 ) = β(N2 ) implica N1 = N2 .
En efecto, si x ∈ N2 , entonces existe y ∈ N1 tal que β(x) = β(y) y por exactitud
existe z ∈ M ′ tal que α(z) = x − y ∈ N2 . Entonces z ∈ α−1 (N2 ) = α−1 (N1 ) y
por lo tanto x − y ∈ N1 . Deducimos que x = y + (x − y) ∈ N1 .

1
Estamos en condiciones de probar la afirmación que hicimos al principio de esta
observación.

n
Lema 2. Si Mi (1 ≤ i ≤ n) son A-módulos noetherianos, también lo es Mi .
i=1

Prueba. Razonamos por inducción sobre n aplicando el lema anterior a la sucesión


exacta
⊕n ⊕
n−1
0 → Mn → Mi → Mi → 0.
i=1 i=1

Solución del ejercicio. Por lo dicho anteriormente, A y A/I1 × · · · × A/Ir son A-módulos.
Consideremos la aplicación

φ : A −→ A/I1 × · · · × A/Ir
a 7−→ (a + I1 , . . . , a + Ir ) .

Es fácil comprobar que φ es un homomorfismo de A-módulos ya que cada proyección natural


A → A/Ii es un homomorfismo de A-módulos. Además, φ es inyectiva pues si a + Ii = 0 + Ii
para todo i = 1, . . . , r entonces a ∈ I1 ∩ · · · ∩ Ir = {0}. Se sigue que A ∼
= A/(0) = A/ ker φ ∼
=
im φ es un submódulo de A/I1 ×· · ·×A/Ir , que es noetheriano, y por lo tanto im φ también.
Como A e im φ son isomorfos, obtenemos el resultado.

Referencias
[AM73] Michael Francis Atiyah and Ian Grant Macdonald. Introducción al álgebra conmu-
tativa. Reverté, 1973. 1

You might also like